Why does the moon appear during the day some of the time and during the night at other times?

Answers

Answer 1

Answer:

As the Earth rotates, the Moon rises just as the Sun sets, but just on that one day of the month. In the days before a full Moon, if you look in the eastern sky, you can find the almost full Moon rising before the sun sets. We can see the moon during the day for the same reason we see the moon at night. The surface of the moon is reflecting the sun's light into our eyes

Explanation:

Hope this helps!

-Liyah<3


Related Questions

Negative charge particle located in the electron cloud is called?

Answers

The negative charge particle located in the electron cloud of an atom is called an electron.

define electron ?

An electron is a subatomic particle that carries a negative electric charge. It is one of the fundamental particles that make up atoms, along with protons and neutrons.

Electrons are extremely lightweight, having a mass of approximately 9.11 x 10^-31 kilograms. They are found in shells or energy levels surrounding the nucleus of an atom and participate in chemical reactions and the flow of electrical current.

The negative charge particle located in the electron cloud of an atom is called an electron.

To learn more about electron follow the given link:

https://brainly.com/question/13998346

#SPJ4

what is the output voltage of a battery if 3 j of energy is expended in moving 0.4 c of charge? round the final answer to one decimal place.

Answers

Voltage measures the electric potential difference between two points in an electric circuit. It is often referred to as electric potential and is measured in volts (V).

The voltage of a battery is an indication of the amount of electrical energy that the battery can provide to a circuit. The output voltage of a battery can be calculated using the formula: V = E/Q; where V is the output voltage, E is the energy expended, and Q is the amount of charge moved. Plugging in the values given in the question, we get:

V = 3 J / 0.4 C

V = 7.5 V

Therefore, the battery's output voltage is 7.5 volts when 3 joules of energy are expended in moving 0.4 coulombs of charge. Rounded to one decimal place, the answer is 7.5 volts.

To learn more about Voltage, visit here

https://brainly.com/question/29445057

#SPJ4

how is the second law demonstrated in the experiment when we use a single fixed mass? group of answer choices the experiment demonstrates the acceleration as being proportional to the force. the experiment shows that that the net force is the sum of forces acting on the object. the experiment shows that the motion of the object stops when the force stops. the experiment shows that the application of force causes the object to acquire a constant speed. that the net force applied is zero. the experiment shows that when a constant force is applied the acceleration increases continuously. the experiment shows that when a force is applied the mass changes inversely with it.

Answers

Experiment demonstrates second law of motion, where acceleration is directly proportional to force and inversely proportional to object's mass.

The second law of motion states that the acceleration of an object is directly proportional to the force applied to it and inversely proportional to its mass. In an experiment where a single fixed mass is used, the second law is demonstrated by the acceleration being proportional to the force applied to the object. As the force acting on the object is increased, the acceleration also increases proportionally. This is because the mass remains constant and the force acting on the object is the net force, which is the sum of all the forces acting on it.

Furthermore, the experiment also shows that the motion of the object stops when the force acting on it stops, and the application of a constant force causes the object to acquire a constant speed. This is because, according to the second law, the object will continue to move with a constant velocity when there is no net force acting on it.

Therefore, the experiment with a single fixed mass is an excellent way to demonstrate the second law of motion, which states that the acceleration is directly proportional to the force applied and inversely proportional to the mass of the object.

Learn more about mass here:

https://brainly.com/question/12323246

#SPJ4

rule stating that the number of transistors ?

Answers

There is no specific rule regarding the number of transistors that can be used in a circuit.

The number of transistors used in a circuit depends on the complexity of the task to be performed, the power requirements of the circuit, and the size and cost constraints. Moore's Law states that a chip's transistor count roughly doubles every two years. In order to improve integrated circuit functionality and performance while lowering costs, the scale goes smaller and the transistor count rises steadily. Moore's Law refers to the fact that the number of transistors on an integrated circuit doubles roughly every two years. According to Moore's Law, a microchip's transistor count doubles every two years. We may expect an increase in the speed and capability of our computers every two years and still pay less for them, according to the law.

To learn more about transistors click here https://brainly.com/question/23822494

#SPJ4

Help now please! I don't really understand the question and would be very grateful for some clarification.

El Toro is the third tallest wooden roller coaster in the world. It has a drop of 55 meters. One empty cart has a mass of 958 kg and 511,000 J of kinetic energy when it reaches the base level of the track. How much energy is converted into heat when the cart is at the base level of the track? Show your work.

Answers

The amount of energy converted into heat when the cart is at the base level of the track is approximately 7490 J.

What is kinetic energy?

The energy an object has as a result of motion is known as kinetic energy. A force must be applied to an object in order to accelerate it.

Kinetic energy:

E = mgh

E = (958 kg) x (9.81 m/[tex]s^2[/tex]) x (55 m) = 5.26 x [tex]10^5[/tex] J

E = KE + Q

KE is the kinetic energy of the cart and Q is the energy converted into heat. Substituting the given value for the kinetic energy, we get:

5.26 x [tex]10^5[/tex] J = 511,000 J + Q

Solving for Q, we get:

Q = 5.26 x [tex]10^5[/tex] J - 511,000 J = 7490 J

Thus, 7490 J energy is converted into heat.

For more details regarding kinetic energy, visit:

https://brainly.com/question/26472013

#SPJ9

When a boxer is moving away from a punch, the force experienced is reduced because______

Answers

When a boxer is moving away from a punch, the force experienced is reduced because C) increased.

What is force of impact of collision?

The force created when things collide is known as the force of impact. The impact or hitting power of your vehicle increases as you increase your speed. The force of impact rises with the square of the increase in speed, according to the rules of physics.

When you throw a punch, you'll apply force to the target by using the momentum that was built during the action and the addition of the snap. This results in impulse (force x time). You may transfer a lot of impulse to the target area and build momentum if you do so.

Therefore, option C is correct.

Learn more about force at:

https://brainly.com/question/12970081

#SPJ1

missing options

A) no different, but the timing is different

B) decreased

C) increased

D) all of the above

the length of the vibrating part of a string on a musical instrument is 0.65 m. when plucked, that string has a fundamental frequency of 110 hz. the string is plucked after the instrument is placed in a room filled with helium gas. what is the wavelength of the sound wave that propagates in the room?

Answers

The wavelength of the sound wave in the room is approximately 3.118 meters, given the frequency and assumed speed of sound.

The speed of sound in a gas relies upon the properties of the gas, and specifically on its temperature, pressure, and sub-atomic weight. Since the issue doesn't give data on these properties, we will accept that the speed of sound in helium is equivalent to in air at room temperature and tension, which is roughly 343 meters each second.

To find the frequency of the sound wave in the room, we can utilize the recipe:

frequency = speed of sound/recurrence

Connecting the qualities we know, we get:

frequency = 343 m/s/110 Hz = 3.118 m

At long last, we can utilize the recipe frequency = speed of sound/recurrence to track down the frequency of the sound wave in the room. The frequency is the distance between two sequential focuses on the wave that are in stage with one another, i.e., that have a similar plentifulness and bearing of movement. For this situation, the frequency ends up being around 3.118 meters.

Subsequently, the frequency of the sound wave in the room is roughly 3.118 meters.

To learn more about wavelength, refer:

https://brainly.com/question/14701011

#SPJ4

a proton and an electron are fixed in space with a separation of 821 nm. calculate the electric potential at the midpoint between the two particles.

Answers

The electric potential at the midpoint between the two particles a proton and an electron is  2.95*10⁻³ V.

You employ the following to determine the two protons' combined electric potential at their midpoint:

[tex]V = V1 + V2 =k\frac{q}{r} +k\frac{q}{r} = 2\frac{kq}{r}[/tex]

where you have considered how each proton contributes to the overall electric potential.

k: Coulomb's constant = 8.98*10^9 Nm^2/C^2

q: charge of the proton = 1.6*10^{-19}C

r: distance from the point (at the midway distance between the protons) to one proton = 821nm = 821nm/2 = 410.5nm = 410.5*10⁻⁹m

You replace the values of the parameters in the equation (1):

[tex]V =2\frac{(8.98*10^-^9 Nm^2/C^2)(1.6*10^-^1^9)}{410.5*10^-^9 m} \\\\V = 2.95*10^-^3 V[/tex]

The electric potential is 2.95*10⁻³ V

Because each proton generates an electric field with a same magnitude but an opposite direction, there is no electric field at the halfway between the protons.

Learn more about electric potential:

https://brainly.com/question/29250185

#SPJ4

what is the electric field vector ~e(r) for r < a, where r is the distance from the center of the shell? explain your answer

Answers

Assuming you are referring to a uniformly charged spherical shell with radius "a", the electric field inside the shell (i.e., for r < a) is zero.

This can be explained by the fact that the shell has a symmetrical charge distribution, and hence, for any point inside the shell, the electric field due to all charges on the shell cancel out exactly. This is a consequence of the superposition principle of electric fields, which states that the total electric field at any point in space is the vector sum of the individual electric fields due to all the charges present. To see why the electric field inside the shell is zero, consider an arbitrary point P inside the shell at a distance "r" from the center of the shell. We can imagine dividing the shell into small elemental areas dA, each of which contains a small amount of charge dQ. The electric field vector due to each of these small charges at point P can be calculated using Coulomb's law as:

[tex]dE = (1 / 4\pi\epsilon _0) (dQ / r^2)r[/tex]

Since the shell is uniformly charged, we can assume that the magnitude of the electric field due to each elemental charge is the same. Also, since the direction of ȓ is different for each elemental charge, the direction of the electric field vector dE varies as we move around the shell. However, due to the symmetry of the charge distribution, the magnitude of the electric field at point P due to all the elemental charges on the shell is the same, and hence the electric field due to all elemental charges is perfectly cancelled out at point P, resulting in a net electric field of zero.

Therefore, we can conclude that the electric field vector ~e(r) for r < a, where r is the distance from the center of the shell, is zero.

To know more about electric field please refer: https://brainly.com/question/8971780

#SPJ4

what is the purpose of the rheostat in this experiment? 1) to limit the current in the circuit. 2) to overheat. 3) as an aid to adjusting the current in the circuit. 4) it is the resistor whose resistance is to going to be determined. group of answer choices 3 and 4 1 and 2 1 and 3 2 and 3 4 only

Answers

The purpose of the rheostat in this experiment is It limits the current in the circuit and as an aid to adjusting the current in the circuit. Option C.

A rheostat is defined as,

A variable resistor which is used for controlling the inflow of electric current either by accelerating or dropping the resistance.

The term rheostat was chased by the English scientist Sir Charles Wheatstone and is concluded from the Greek word “ rheos ” and “ statis ” which means current controlling device.

All electrical circuit has three fundamental components, and they are

The circuit's applied voltage

through the circuit current

the circuit's resistance offering

A rheostat is a variable resistor that's used to control the inflow of electrical current in a circuit. By conforming the resistance of the rheostat, it's possible to acclimate the quantum of current flowing through the circuit.

This can be useful in a variety of electrical trials and operations, similar as testing the current- voltage relationship of a circuit or regulating the quantum of power supplied to a cargo.

Learn more about rheostat;

https://brainly.com/question/30620580

#SPJ4

This is a GCE physics past paper question. Can someone please give answer and also give reasoning.

Answers

Answer:

60mV

Explanation:

But I'm not so sure, i tried

a 2-khz sound wave traveling through a bar of iron has a wavelength of 2.56 meters. what is the velocity of sound in iron?

Answers

From the information provided, the velocity of sound in iron is approximately 5120 m/s.

Velocity is a vector quantity that describes the rate of change of an object's position with respect to time. It is defined as the displacement of an object divided by the time interval over which the displacement occurred, and it has both magnitude and direction.

We can use the formula for the velocity of a sound wave:

v = fλ

where v is the velocity of the sound wave, f is the frequency of the wave, and λ is the wavelength of the wave.

Substituting the given values, we get:

v = (2 kHz)(2.56 m) = 5120 m/s

Learn more about velocity here: brainly.com/question/24445340

#SPJ4

if the field changes in value from -0.100 t to 0.150 t in an interval of 0.500 s, what is the magnitude of the average voltage induced in the coil?

Answers

The magnitude of the average voltage induced in the coil if the field changes in value from -0.100 t to 0.150 t in an interval of 0.500 s is 125000 V.

The "pressure" that pushes electricity is referred to as voltage. A voltage is measured in volts (V), and greater voltages result in more electricity flowing to an electronic equipment. Yet, electronic gadgets must work within a range of voltages since too much voltage might harm their circuitry.

Change in magnetic field = 0.150 - (- 0.1)

= 0.250 T

Area of coil = 500 m²

Induced emf = N[ change in flux]/time

flux = Area x magnetic field

Initial flux = 500 x 0.150 = 75 = 75  Tm²

Final flux = 500 x (-1.00) = 50 = - 50 Tm²

Change in flux = 75 + 50 = 125

Induced emf = 500(125) / 0.5

= 62500 / 0.5

= 125000 V

The average voltage is 125000 V

In contrast, a voltage that is too low can also be problematic since it prevents circuits from functioning and renders the devices that are constructed around them ineffective. To handle electronic devices properly and pinpoint the root causes of troubles when they arise, one must have a working knowledge of voltage and how to resolve related problems.

Learn more about Average voltage;

https://brainly.com/question/14015406

#SPJ4

When a boxer moves into an oncoming punch, the force experienced is
answer choices
decreased
increased
no different, but the timing is different
all of the above

Answers

It lengthens the impact window while lessening the force of impact. The moment of impact is shortened and the energy of impact is increased when the boxer moves toward the blow. Thus, option B is correct.

What is the force experienced by  boxer moves?

The force felt by a boxer as they move away from a blow is lessened since the punch has more time to make contact with them. Impulse is the result of the force and the moment of application, and this notion explains why “rolling with the punches” in boxing is effective.

Since force is inversely proportional to time, the effect of force is diminished when he moves his head rearward since time increases. Hence, the force lessens and the boxer impacts somewhat. Q.

Therefore, When a boxer moves into an oncoming punch, the force experienced is increased

Learn more about force here:

https://brainly.com/question/13191643

#SPJ4

what is the conversion factor between cm2 and m2? 0.01 m2/cm2 10 m2/cm2 10000 m2/cm2 100 m2/cm2 0.0001 m2/cm2

Answers

The conversion factor between square centimeters ([tex]cm^2[/tex]) and square meters ([tex]m^2[/tex]) is [tex]0.0001 m^2/cm^2.[/tex]

to convert a given area from square centimeters to square meters, you need to multiply it by 0.0001, and to convert it from square meters to square centimeters, you need to multiply it by 10,000.

This conversion factor is derived from the fact that there are 100 centimeters in a meter, and the conversion from one unit to another involves squaring the length measurement. This means that the conversion factor between the area in square centimeters and square meters is [tex](1 cm / 100 cm)^2[/tex] = [tex]0.0001 m^2/cm^2[/tex].

It's important to be able to convert between different units of area, as different applications may require different units. For example, when measuring the area of a small object, square centimeters may be a more appropriate unit, while for larger areas, square meters may be more suitable.

Learn more about conversion factor here:

https://brainly.com/question/23718955

#SPJ4

An object traveling in the negative direction and accelerating in the negative direction will slow down. Group of answer choices

True
False

Answers

False. An object traveling in the negative direction and accelerating in the negative direction will not slow down, but will actually speed up.

Acceleration is the rate of change of velocity, and if an object is accelerating in the negative direction, it means that its velocity is decreasing in the positive direction and increasing in the negative direction. Therefore, the object will continue to move in the negative direction at a faster speed.

It is important to consider that acceleration is a magnitude that can cause the velocity of a body to increase or decrease.

See more about acceleration at https://brainly.com/question/460763.

#SPJ11

Express all answers in scientific notation and state their units clearly Given D = M/V Find D When M = 2.5kg V =50m³​

Answers

Answer:

In scientific notation, the density would be 5 x 10^-2 kg/m³ or 5 x 10^-2 g/cm³.

Explanation:

To find D, we just need to divide M by V.

M = 2.5 kg

V = 50 m³

D = M/V = 2.5 kg / 50 m³ = 0.05 kg/m³

The unit of density (D) is kg/m³, which can also be expressed as g/cm³.

In scientific notation, the density would be 5 x 10^-2 kg/m³ or 5 x 10^-2 g/cm³.

the wheelbase on a truck is 2.4 m wide and the truck's center of mass is located along the vertical centerline of the truck and 2.0 m above the bottom of the tires. the truck is going around a banked turn, when it is forced to stop. what is the maximum slope that the bank can have such that the truck will not tip over?

Answers

As center of mass is located along the vertical centerline of the truck, The maximum slope that the bank can have such that the truck will not tip over is 16.7°.

To calculate the maximum slope that the bank can have such that the truck will not tip over, we need to consider the forces acting on the truck and the torque due to these forces. The forces acting on the truck are the weight of the truck and the normal force of the road, and the torque is due to the fact that these forces do not act through the center of mass of the truck. The maximum slope can be calculated by finding the angle at which the normal force is reduced to zero, causing the truck to tip over. At this point, the weight of the truck will provide the only force acting on the truck, and it will act through the edge of the tires.

Let θ be the angle of the bank, and let W be the weight of the truck. The normal force acting on the truck:-

N = W cosθ

The weight of the truck acts through the center of mass, which is located 2.0 m above the bottom of the tires. The torque due to the weight:-

τ W = W * 2.0 * sinθ

The normal force acts through the center of the tires, which are located 1.2 m apart. The torque due to the normal force:-

τ N = N * 1.2/2 * sinθ

For the truck not to tip over, the torque due to the normal force must be greater than or equal to the torque due to the weight:

τ N ≥ τ W

Reserving the expressions for N and the torques:-

W cosθ * 0.6 * sinθ ≥ W * 2.0 * sinθ

Simplifying:-

tanθ ≥ 0.6/2.0

θ ≥ arctan0.3

θ ≥ 16.7°

Therefore, the maximum slope that the bank can have such that the truck will not tip over is 16.7°.

To know more about mass please refer: https://brainly.com/question/30259066

#SPJ4

in the figure, a 5.00-kg block is moving at 5.00 m/s along a horizontal frictionless surface toward an ideal massless spring that is attached to a wall. after the block collides with the spring, the spring is compressed a maximum distance of 0.68 m. what is the speed of the block when it has moved so that the spring is compressed to only one-half of the maximum distance?

Answers

When a 5.0 kg block moves at 5.0 m/s over a horizontal frictionless surface towards an ideal spring attached to a wall, the maximum speed of the block is 4.33 m/s when the spring is squeezed to one-half of the maximum distance.

According to the law of conservation of energy, the kinetic energy of the mass equals the work done on the spring.

i.e.

1/2 mv² = 1/2 kx²max

In light of this:

The weight of the block is 5.0 kg.

the speed at which it is travelling = 5.0 m/s spring

compression = 0.68 m

By multiplying both sides of the equation by 2, we get:

mv² = kx²max

Making (k) the formula's subject;

k = mv²/ x²max

k = 5*5²/0.68²

k = 125/0.4624

k = 268.17 N/m

However, when the distance is reduced to one-half of its maximum, the speed is calculated as follows:

x = 0.68/2 = 0.34 m

1/2 mv² - 1/2 mv² = 1/2 kx²

m(v²₀-v²) = kx²

(v²₀ - v²) = kx²/m

(5² - v²) = 268.17 * 0.34²/5.0

25-v² = 6.2

v² = 25.625

v² = 18.75

v = √18.75

v = 4.33 m/s

As a result, we may deduce that the block's speed when the spring is squeezed to only one-half of its maximum distance is 4.33 m/s.

Learn more about kinetic energy from here;

https://brainly.com/question/26472013

#SPJ4

a car moving a constant 7.0 m / s turns a corner with a radius of 20 m . what is the acceleration of the car?

Answers

The acceleration of the car as it turns the corner is 2.45 m/s^2, and it is directed towards the center of the turn.

What is Acceleration?

When an object changes its velocity, either by increasing or decreasing its speed or by changing its direction of motion, it experiences acceleration.

The formula for acceleration is:

a = (v2 - v1) / t

where a is the acceleration, v2 is the final velocity, v1 is the initial velocity, and t is the time taken for the change in velocity to occur.

To find the acceleration of the car as it turns the corner, we can use the centripetal acceleration formula:

a = v^2 / r

where a is the acceleration, v is the velocity of the car, and r is the radius of the turn.

In this case, the velocity of the car is constant at 7.0 m/s, and the radius of the turn is 20 m.

a = (7.0 m/s)^2 / 20 m

a = 2.45 m/s^2

Therefore, the acceleration of the car as it turns the corner is 2.45 m/s^2, and it is directed towards the center of the turn.

Learn more about Acceleration from given link

https://brainly.com/question/605631

#SPJ1

you're flying a cessna 182 skylane, n935fa, on a cross-country vfr flight with flight following. during a handoff, the ideal call sign to use when contacting the new controller is .

Answers

You're flying a Cessna 182 Skylane, N935FA, on a cross-country VFR flight with flight following. During a handoff, the ideal call sign to use when contacting the new controller is Skylane 935FA.

Flight following is a relatively simple concept as it’s an aircraft flying under VFR that is taking use of Air Traffic Control (ATC) services. Practically, it indicates that several advisories may be available from the controller and the flight is radar identified by ATC.

The service is given on a workload-permitting basis and involves multiple layers of service, there’s not entirely uniform delivery among Center or Approach control facilities. The fusion of these realities has resulted in misinformation, misunderstandings, and even no awareness of its existence.

To learn more about VFR flight,

brainly.com/question/14308163

#SPJ4

a shiny sports car at the top of a vertical cliff has a potential energy of 100 mj relative to the ground below. unfortunately, a mishap occurs and it falls over the edge. when it is halfway to the ground, its kinetic energy is

Answers

The car has 50,000,000 J of kinetic energy when it's halfway to the ground.

What is kinetic energy?

The energy that an object has as a result of motion is known as kinetic energy. Kinetic energy, which may be transferred to other objects or changed into other kinds of energy, is present in every moving item.

According to question:

At the top of the cliff, the total energy of the car is purely potential energy, which is given by:

PE = mgh

where m is the mass of the car, g is the acceleration due to gravity (approximately 9.81 m/s²), and h is the height of the cliff relative to the ground.

In this case, the potential energy of the car is given as 100 MJ. However, we need to convert this to joules in order to use it in the kinetic energy calculation later.

1 MJ = 1,000,000 J, so

PE = 100 MJ = 100,000,000 J

Now, when the car falls halfway to the ground, it has lost half of its potential energy, which means it now has:

PE = 0.5 * 100,000,000 J = 50,000,000 J

At this point, the car has also gained some kinetic energy due to its motion. The total energy (potential + kinetic) must still be conserved, so we can use the law of conservation of energy to find the kinetic energy of the car when it's halfway to the ground:

PE(initial) = KE(final)

where KE is the kinetic energy of the car at the final point (when it's halfway to the ground).

So, substituting the values we have:

50,000,000 J = KE

We now know that the car has 50,000,000 J of kinetic energy when it's halfway to the ground.

To know more about kinetic energy visit:

https://brainly.com/question/26472013

#SPJ4

2. a student blows across the top of an empty bottle causing it to resonate at its fundamental frequency. the bottle has a height of .4m. the speed of sound is 340 m/s. what is the fundamental frequency? (4 pts)

Answers

The fundamental frequency of the resonating air column in the bottle is 212.5 Hz. When a tube or bottle with one closed end is excited, it can resonate at certain frequencies, with the lowest frequency being the fundamental frequency.

The fundamental frequency of a resonating air column in a tube with one closed end, like the bottle in this case, is given by:

f = v/4L

where f is the fundamental frequency, v is the speed of sound, and L is the length of the air column.

In this case, the length of the air column is equal to the height of the bottle, which is 0.4 meters. The speed of sound is 340 m/s. Substituting these values into the formula, we get:

f = (340 m/s)/(4 x 0.4 m) = 212.5 Hz

Therefore, the fundamental frequency of the resonating air column in the bottle is 212.5 Hz.

When a tube or bottle with one closed end is excited, it can resonate at certain frequencies, with the lowest frequency being the fundamental frequency. The length of the air column inside the tube determines the fundamental frequency of the resonating air column. The resonant frequencies of the tube are determined by the speed of sound in air, the length of the air column, and the properties of the tube, such as its shape and the position of the closed end.

The fundamental frequency of a resonating air column in a tube with one closed end can be determined using the formula:

f = v/4L

where f is the frequency, v is the speed of sound, and L is the length of the air column. This formula assumes that the air column is a simple tube with one closed end, and that the air inside the tube is at rest before the sound wave is introduced.

Learn more about frequency here:

https://brainly.com/question/9424813

#SPJ4

a 0.25 kg ball is thrown with an initial velocity of 32 m/s at an angle of 30o. determine its total energy at the top of its flight path.

Answers

The total energy of the ball at the top of its flight path is 224.2 Joules.

What is the total energy?

To determine the total energy of the ball at the top of its flight path, we need to consider the ball's kinetic energy and potential energy.

First, let's find the maximum height the ball will reach. At the top of its flight path, the ball will have no vertical velocity, so we can use the equation:

[tex]v_f^2 = v_i^2 + 2ad[/tex]

where[tex]v_f[/tex]is the final velocity (0 m/s), [tex]v_i[/tex] is the initial velocity (32 m/s), a is the acceleration due to gravity [tex](-9.8 m/s^2)[/tex], and d is the maximum height. Solving for d, we get:

[tex]d = (v_f^2 - v_i^2)/(2a) = (0 - (32 m/s)^2)/(2*(-9.8 m/s^2)) = 51.8 m[/tex]

Next, let's find the kinetic energy of the ball at the top of its flight path. Since the ball has no vertical velocity at this point, we can use the horizontal velocity to find the kinetic energy. The horizontal velocity is:

[tex]v_x = v_i cos(30o) = 27.7 m/s[/tex]

The kinetic energy is:

[tex]K = (1/2)mv_x^2 = (1/2)(0.25 kg)(27.7 m/s)^2 = 96.4 J[/tex]

Finally, let's find the potential energy of the ball at the top of its flight path. The potential energy is:

U = mgh = [tex](0.25 kg)(9.8 m/s^2)(51.8 m)[/tex] = 127.8 J

The total energy at the top of the ball's flight path is the sum of the kinetic and potential energy:

E = K + U = 96.4 J + 127.8 J = 224.2 J

Therefore, the total energy of the ball at the top of its flight path is 224.2 Joules.

Learn more about energy, here:

https://brainly.com/question/1932868

#SPJ1

what sort of human activity do you think is contributing to increase carbon dioxide

Answers

The human activities that are contributing to increase carbon dioxide are said to be releasing chemicals into the atmosphere, deforestation, construction, burning fossil fuels, etc.

Burning the fossil fuels is done to produce coal, generation of electricity, in factory usage which results in the release of huge amounts of CO₂.

Smoke from the vehicles contribute to the increased CO₂ levels.

Deforestation is done to acquire land for agriculture and construction purposes. Urbanisation leads to increase in CO₂ levels with the increased construction.

Construction of high rise buildings leads to the usage of a lot of cement. Thus increasing CO₂ levels into the atmosphere.

Increased CO₂ levels in the atmosphere leads to the climate change by increasing the temperature of the earth.

To know more about human activities:

https://brainly.com/question/30665396

#SPJ4

once a standing wave appears in the string with several nodes, what would happen if you were to carefully hold the string with narrow tweezers at a node (a geometric point)? will the standing wave pattern change or be destroyed?

Answers

Standing waves are most often caused by the resonance phenomenon, which happens when waves are reflected back and forth at the resonator's resonant frequency and interfere, causing standing waves to form inside the resonator.

What cause the standing wave pattern change?

The observed wave pattern contains points that appear to be standing still, which is why the pattern is typically referred to as a standing wave pattern.

Such patterns can only be produced by specific vibrational frequencies within the medium. The term "harmonics" or "only harmonics" is used to describe these frequencies.

A reflective boundary of some kind is required for a standing wave to form. The shape is then affected by the wave's frequency, the waveguide's length, and the boundary's acoustic impedance.

Therefore, standing wave pattern change.

Learn more about standing wave here:

https://brainly.com/question/28191139

#SPJ4

What is the activation energy of a reaction if it has the following rate constants?
Rate Constant Temperature
6.20 x 10^-4 s^-1 700 K
2.39 x 10^-2 s^-1 760 K

Answers

The activation energy of a reaction if it has the following rate constants is calculated to be 126.8 kJ/mol.

To calculate the activation energy of a reaction, we can use the Arrhenius equation:

k = A × e^(-Ea/RT)

where,

k is the rate constant

A is the pre-exponential factor

Ea is the activation energy

R is the gas constant

T is the temperature in Kelvin

We have two rate constants at different temperatures, so we can set up two equations:

k₁ = A × e^(-Ea/RT₁)

k₂ = A × e^(-Ea/RT₂)

We want to solve for Ea, so we can take the natural logarithm of both sides of each equation:

ln(k₁) = ln(A) - Ea/RT₁

ln(k₂) = ln(A) - Ea/RT₂

We can subtract the second equation from the first to eliminate ln(A):

ln(k₁) - ln(k₂) = Ea/R × (1/T₂ - 1/T₁)

Now we can solve for Ea:

Ea = -R × (ln(k₁) - ln(k₂)) / (1/T₂ - 1/T₁)

Plugging in the given values, we get:

Ea = -8.314 J/mol/K × (ln(6.20 × 10⁻⁴) - ln(2.39 × 10⁻²)) / (1/760 K - 1/700 K)

Ea ≈ 126.8 kJ/mol

Therefore, the activation energy of the reaction is approximately 126.8 kJ/mol.

To know more about activation energy:

https://brainly.com/question/29801443

#SPJ4

assuming the lower arm has a mass of 2.8 kg and its cg is 12 cm from the elbow-joint pivot, how much force must the extensor muscle in the upper arm exert on the lower arm to hold a 7.5 kg shot put (fig. 12-7)?

Answers

The correct option is A, 1000N force must the extensor muscle in the upper arm exert on the lower arm to hold a 7.5 kg shot put.

Mm = mass of muscle

Me = mass of elbow

Mb = mass of ball

τ = Mad-> torque = mass * gravity * distance

-Mmad + Mead + M2ad = 0

Mm (9.80) (-.025m) + (2.8kg) (9.80) (.12m) + (7.5kg) (9.80) (.3m) = 0

Mm = 103.44kg

F = Ma = (103.44kg) (9.80)

= 1014N ~ 1000N

Force is a physical quantity that describes the interaction between objects or systems. It can be defined as the push or pull on an object resulting from the interaction between two or more bodies. The unit of force is the newton (N), and it is represented by the symbol F.

Force can change the motion of an object by accelerating,decelerating, or changing its direction. The magnitude and direction of a force are critical in determining how it will affect an object. There are several types of forces, including contact and non-contact forces. Contact forces involve direct physical contact between objects, while non-contact forces occur at a distance without physical contact.

To learn more about Force visit here:

brainly.com/question/30507236

#SPJ4

Complete Question: -

Assuming the lower arm has a mass of 2.8 kg and its CG is 12 cm from the elbow-joint pivot, how much force must the extensor muscle in the upper arm exert on the lower arm to hold a 7.5 kg shot put (Fig. 12-7)? ?????

A. 1000 N

B. 1500 N

C. 100 N

D. 500 N

E. 750N

3. The electric field at a point 27 cm from a charged particle is 105 N
C
toward the particle.
What is the charge on the particle?

Answers

The particle has a charge of 2.44 x 10-8 C. when electric field at a point 27 cm from a charged particle is 105 N

What is charge?

The electrical properties of particles like electrons and protons are described by charge, a fundamental property of matter. Particles with the same electric charge repel one another while those with the opposite charge attract one another. Electric charge can be positive or negative. The Coulomb (C) is the charge measurement unit, and particles' charges are typically measured in Coulombs.

E is equal to kQ/r₂, where k is the Coulomb constant, r is the distance from the particle to the point where the electric field is being measured, Q is the charge on the particle, and E is the electric field.

The electric field at a distance of 27 cm from the particle is calculated to be 105 N/C in the particle's direction. We now have:

By substituting these values into the equation for the electric field, we obtain: E = 105 N/C r = 27 cm = 0.27 m

105 N/C = (8.99 x 10⁹ N m²/C²) × Q / (0.27 m)²

Solving for Q, we get:

Q = (105 N/C) × (0.27 m)² / (8.99 x 10⁹ N m²/C²)

Q = 2.44 x 10⁻⁸ C

Therefore, the charge on the particle is 2.44 x 10⁻⁸ C.

Learn more about charge:

brainly.com/question/25923373

#SPJ1

at what distance (in mm) from a long, straight wire carrying a current of 16 a will the magnitude of the magnetic field be 1.2 mt?

Answers

The distance from the wire at which the magnetic field has a magnitude of 1.2 mt is 26.7 mm.

What is magnetic field?

A region of space where the magnetic forces of a magnet or a moving electric charge can be felt is called a magnetic field. Moving electric charges, such the spinning electrons in an atom or the electrons in a wire, create magnetic fields. They can also be created by the Earth's core or permanent magnets.

The strength and direction of magnetic fields define them. A magnetic field's direction can be determined by calculating the force it would apply to a compass's north pole. The amount of force that a magnetic field would apply to a charged particle travelling through the field provides an indication of its intensity.

According to question:

To calculate the distance at which the magnetic field has a magnitude of 1.2 mt from a long, straight wire carrying a current of 16 A, we can use the formula for the magnetic field of a current-carrying wire:

B = (μ0 / 2π) * (I / r)

where B is the magnetic field, I is the current, r is the distance from the wire, and μ0 is the permeability of free space (4π x 10^-7 T·m/A).

Rearranging the formula to solve for r, we get:

r = (μ0 / 2π) * (I / B)

Plugging in the given values, we get:

r = (4π x 10^-7 T·m/A / 2π) * (16 A / 1.2 x 10^-3 T)

r = 2.67 x 10^-5 m = 26.7 mm

To know more about magnetic field visit:

brainly.com/question/14848188

#SPJ4

Other Questions
Which pairs of rectangles are similar polygons?Select each correct answer. mention the two issues that greatly raised the political stakes were? a patient is admitted to the critical care unit with an anion gap of 24 meq/l. this laboratory finding is characteristic of which condition? what 3 literary devices does the director use to communicate the characterization of both batman and joker Please help! I don't understand how my teacher does the work, so therefore, I don't understand how to do these word problems. 100-point jackpot for answering. The table below shows the cost of mailing a postcard in different years. Duringwhich time interval did the cost increase at the greatest average rate? A right square pyramid has an altitude of 10 and each side of the base is 6. To the nearest tenth of a centimeter, what is the distance from the apex, or top of the pyramid, to each vertex of base? the nurse is auscultating the lungs of a lethargic, irritable 6-year-old boy and hears wheezing. the nurse will most likely include which teaching point if the child is suspected of having asthma? Select the correct answer. Simplify the following expression. (4x + 7)2 a. 16x2 - 49 b. 16x2 + 56x + 49 c. 16x2 + 28x + 49 d. 16x2 + 49. as a philosophy for business ethics, utilitarianism focuses attention on the need to Binomial distribution calculationIf n=5 and p=0.7, find P(x=4)Give at least 4 decimal places. Microsoft Windows XP operating system is installed on 20% of the company's computers, Microsoft Windows 7 is installed on 85% of computers, and Linux operating system is installed on 10%. At the same time, Linux and Microsoft Windows 7 are installed on 6% of computers, Microsoft Windows XP and Linux on 4%, all three programs are installed on 2% of computers. How many percent of computers have the Microsoft operating system installed? Stability, clear expectations, and creating order are all strengths of bureaucracies. True or False? Let X denote the amount of time a book on two-hour reserve is actually checked out, and suppose the cdf is the following. F(x) = 0 x < 0 x2 25 0 x < 5 1 5 x Use the cdf to obtain the following. (If necessary, round your answer to four decimal places.)(a) Calculate P(X 3).(b) Calculate P(2.5 X 3).(c) Calculate P(X > 3.5).(d) What is the median checkout duration ? [solve 0.5 = F()].(e) Obtain the density function f(x). f(x) = F '(x) =(f) Calculate E(X).(g) Calculate V(X) and x. V(X) = x =(h) If the borrower is charged an amount h(X) = X2 when checkout duration is X, compute the expected charge E[h(X)]. What is the name of the approach that offers a line-by-line analysis of an excerpt from a play?O Acomparison/contrastsO B.analysisO C.explication de texteO D. academic argument apply conditional formatting to the selected cells using the red gradient fill data bar. Next > Pretest: A Nation Emerges (1945-1960)6Select the correct answer from each drop-down menu.How was the United States impacted after World War II? When solving an inequality, if you the coefficient is negative, as in-3x > 9, the inequalityTrueFalse what arguments does fitzhugh use to promote slavery? what basic premise underlies his ideas? can you think of a modern parallel to fitzhughs argument? auto = "self"crat/cracy = "rule"dem = "peopla"Which sentence uses the underlined word correctly? As the sole decision maker, the CEO considered himself the democratic leader of his company. Since the leader consolidated all the power for his own use, the country is now essentially an autocracy The solitary queen ruled the democracy with an iron fist and allowed no interference with her decisions. The autocratic government was particularly popular for its habit of implementing citizens' suggestions.